Questions tagged [multivariable-calculus]

Use this tag for questions about differential and integral calculus with more than one independent variable. Some related tags are (differential-geometry), (real-analysis), and (differential-equations).

Filter by
Sorted by
Tagged with
0 votes
0 answers
16 views

Local maxima of $f(x,y) = x^2 + y^2 - xy$ subject to $x^2 = 16$, $y^2 = 16$

Given the problem below: Write down the Lagrangian when the problem is to maximise the function $$ f(x, y)=x^2+y^2-x y $$ on the set $$ \{(x, y):-4 \leq x \leq 4,-4 \leq y \leq 4\} . $$ Use the Kuhn-...
xolotl123's user avatar
0 votes
0 answers
21 views

Tangent Vector, Principal normal vector, Binormal vector, and Torsion

So I'm trying to fully grasp how all these relate. My current understanding is that the tangent vector describes the direction in which the curve is going/curving. Meanwhile, the principal norm is ...
A Student 's user avatar
1 vote
1 answer
16 views

Confusion about the units of divergence

For a 3-D vector field $V = (u \hat{\textbf{i}} + v\hat{\textbf{j}} + w\hat{\textbf{k}})$: $$\nabla \cdot V\Big[\frac{m}{s}\Big] = \frac{\partial u [\frac{m}{s}]}{\partial x [m]} + \frac{\partial v[\...
Researcher R's user avatar
0 votes
0 answers
60 views

Needing help re-deriving an equation in a paper

I am trying to understand where an equation came from in a paper. Earlier, the paper gives these relations $$\left(\nabla^2-\frac{1}{r^2} \right) u_r +\frac{1}{1-2\nu} \frac{\partial e}{\partial r} + ...
Leenda's user avatar
  • 11
1 vote
0 answers
37 views

Show equivalence of two very long integrals.

I'm trying to show that the following integral $$\int_0^\infty \int_{-\infty}^s f(x \vee s-b) \sqrt{\frac{2}{\pi}}\frac{1}{t^{\frac{3}{2}}}(2s-b)e^{-\frac{(2s-b)^2}{2t}-\mu(b+\frac{\mu t}{2})}db ds$$ ...
user82832's user avatar
1 vote
0 answers
13 views

Logistic regression with gradient descent derivation

Our maximum likelihood is: $$l(\beta)=\sum_{i=1}^{n}y\log(p(x))+(1-y)\log(1-p(x))$$ $$l(\beta) = \sum_{i=1}^{n} y^{(i)} \log(\sigma(\beta^\intercal x^{(i)})) + (1 - y^{(i)}) \log(1 - \sigma(\beta^\...
samsamradas's user avatar
-1 votes
0 answers
28 views

Some Confusion with the Partial Derivative Chain Rule

In my calculus class, we were asked to show that in polar coordinates, Laplace's equation is: $$ \dfrac{\partial^2z}{\partial x^2} + \dfrac{\partial^2z}{\partial y^2} = 0 = \dfrac{\partial^2z}{\...
R. Modi's user avatar
1 vote
1 answer
81 views

Finish this proof of gradient in polar coordinates

In my exercise, we want to derive the formula of the gradient in polar coordinates. We end up showing that $$\nabla f(r\cos \theta , r\sin\theta)=\frac{ \partial g }{ \partial r }(r,\theta)e_{r }+\...
KiwiKiwi's user avatar
  • 169
0 votes
0 answers
13 views

Differentiable function and zero derivative [closed]

I'm trying to solve the following question: Consider a differentiable function $f: U \subset \mathbb{R}^{n} \longrightarrow \mathbb{R}$, where $U$ is open and connected. The differential $ df(p): \...
ganom51's user avatar
  • 21
0 votes
5 answers
75 views

Help with integration over a triangular region.

I'm currently trying to wrap my head around double integrals over a triangular region, when you are given the vertices of the triangle. I need to do the integral $$ \iint_D 2e^{-y-x} \,\mathrm{d}y\,\...
HMND's user avatar
  • 1
0 votes
0 answers
40 views

What integral is used to calculate the electric field generated by a continuous charged curve?

I'm studying Multivariable Mathematics, by Ted Shifrin, in which one reads that ''the gravitational force exerted by a continuous mass distribution $\Omega$ with density function $\delta$ is $$\mathbf{...
Henrique Fonseca's user avatar
0 votes
1 answer
56 views

A question about a multivariable limit

I have $$\lim_{(x,y)\to (1,0)} \frac{xy^2 -y^2}{(x-1)^2+3y^4}$$ where the answer must be "the limit does not exist". I set $y=m(x-1)$ and get $$\lim_{x\to 1} \frac{m^2(x-1)}{1+3m^4(x-1)^2}=0....
Ninja's user avatar
  • 2,807
0 votes
1 answer
29 views

How to write Jacobian matrix in more vector-like format

Let $\mathbf{f}: \mathbb{R}^{n} \rightarrow \mathbb{R}^{m}$ be a vector field. Suppose I wish to compute $\frac{\partial \mathbf{f(x)}}{\partial \mathbf{x}}$. Then we have the Jacobian. $$\frac{\...
Stan Shunpike's user avatar
1 vote
0 answers
24 views

How does the del operator work exactly? [duplicate]

In Cartesian coordinates, vector operations are as simple as if we were treating the del operator as like a vector. $$\nabla = \Big(\frac{\partial}{\partial x}\hat{\textbf{i}} + \frac{\partial}{\...
Researcher R's user avatar
0 votes
0 answers
18 views

How to solve simple second order ODE with RHS $x / \sigma^2$

I have the following Hamiltonian systems for $i=1, \ldots, d$ and $\sigma_i > 0$ $$ \begin{align} \frac{d}{dt} x_i &= v_i \\ \frac{d}{dt} v_i &= \frac{x_i}{\sigma_i^2} \end{align} $...
Physics_Student's user avatar
0 votes
1 answer
23 views

Length of a regular arc

I have to prove that if two arc are (positive) equivalent, they have the same length. Let $\alpha:[a,b]\to \mathbb{R}^n$ and $\beta:[c,d]\to \mathbb{R}^n$ two differentiable arcs and let $h:[a,b]\to [...
Sigma Algebra's user avatar
0 votes
1 answer
51 views

Does limit exists or not?

I've been trying to solve with trajectories, but I haven't been able to find an answer. $\lim_{(x,y)\to (0,0)} \frac{x^2y^4}{2x^2-2xy+2y^4}$
Ever David Osorio's user avatar
0 votes
1 answer
39 views

Double integral $\int \int_R \sqrt{1-x^2-y^2} \mathrm{d}x\mathrm{d}y$ (from the surface integral $\int \int_S z^2 \mathrm{d}S$)

I have this double integral over a region $R\in \mathbb{R}^2$, which is the projection of the surface $S: \{x^2+y^2+z^2=1| \; x\geq0,y\geq0,z\geq0\}\in \mathbb{R}^3$ (located in the 1st quadrant). I ...
Vebjorn's user avatar
  • 179
5 votes
3 answers
305 views

Interpreting a notation in calculus of variations (differentiating with respect to a derivative)

Consider a functional $J[y]$ defined by: $$J[y] = \int_a^b F(x, y, y') dx \tag{1}$$ Here, $F$ is a function that depends on the independent variable $x$, the function $y(x)$, and its derivative $$y' = ...
AChem's user avatar
  • 317
0 votes
1 answer
35 views

A question about rudin 9.32 Why $PA=A$

9.32 Theorem: Suppose 𝑚,𝑛, are nonnegative integers, $𝑚≥𝑟$,$𝑛≥𝑟$,$𝐹$ is a $𝐶^{1}$ mapping of an open set 𝐸⊂$𝑅^{𝑛}$ into $𝑅^{𝑚}$, and $𝐹′(𝑥)$ has rank $r$ for every $𝑥∈𝐸$. Fix $𝑎∈𝐸$...
Mathematics enjoyer's user avatar
1 vote
1 answer
46 views

Integrate a sum of trig function under absolute value

Let $n \in \mathbb{N}$, I'm trying to compute en explicit formula for the following integral: $$ \operatorname{I}\left(n\right) = \int_{\left[0,2\pi\right]^{n}} \left\vert\cos\left(x_1\right) + \cdots ...
MathRevenge's user avatar
0 votes
0 answers
51 views

Is the implicit function unique?

Does the equation $$xy^2+xz^3+\ln z=0$$ define the unique implicit function $z=g(x,y)$ in the neighborhood of $(0,1)$? If yes, compute $\dfrac{dz}{dx}(0,1)$ and $\dfrac{dz}{dy}(0,1).$ My attempt: Let ...
lee max's user avatar
  • 436
0 votes
1 answer
45 views

Change of coordinates on $\nabla(h\circ\varphi^{-1})$ where $h,\varphi:\mathbb{R}^n\to\mathbb{R}^n$

Say I have a smooth vector-valued function $h:\mathbb{R}^n\to\mathbb{R}^n$ and a smooth diffeomorphism $\varphi:\mathbb{R}^n\to\mathbb{R}^n$. Consider the gradient of the composition $h\circ\varphi^{-...
Stuck's user avatar
  • 1,714
0 votes
1 answer
41 views

functionally unfair coins

If the faces of a two-sided "coin" are parameterized by (convex in the direction of heads or tails, respectively) continuous functions $f_1(r,\theta)$ and $f_2(r,\theta)$ (of two variables: ...
vallev's user avatar
  • 90
1 vote
1 answer
70 views

Is this function differentiable? (Error in the notes?)

So I'm studying in my notes, and there is this "example" (not worked) where it's stated that the following functions is NOT differentiable at $(0, 0)$: $$f(x, y) = \begin{cases} \frac{\sqrt{...
Heidegger's user avatar
  • 3,255
1 vote
1 answer
89 views

Approximation of a non differentiable function

Can you please tell me if/where I am wrong about this? Let $f: \mathbb{R}^2\to \mathbb{R}$ a function. Let $f$ be derivable at $\left( \frac{6}{5}, \frac{6}{5}\right)$ but not differentiable at that ...
Heidegger's user avatar
  • 3,255
0 votes
1 answer
24 views

Is there any way to compact the propagation of uncertainty formula in terms of vectors?

The uncertainty associated to $\xi=f(\mathbf x)$ with $\mathbf x\in\mathbf R^n$ is $$\delta\xi=\sqrt{\sum_{i=1}^n\left(\frac{\partial \xi}{\partial x_i}\delta x_i\right)^2}$$ What I was wondering was ...
Joan S. Guillamet F.'s user avatar
0 votes
0 answers
26 views

The chain rule for partial derivatives

My goal is to understand a statement about partial derivatives made in the appendix of a textbook about manifolds. From the book: Let $U \subseteq \mathbb{R}^n$ and $\tilde{U}\subseteq \mathbb{R}^m$ ...
Maple's user avatar
  • 1
0 votes
0 answers
33 views

Find an explicity formula for the inverse of the function $\textbf{f}$ given by $f_1=e^x\cos y$ and $f_2=e^x\sin y$ [duplicate]

Let $\textbf{f}$ be given by $f_1=e^x\cos y$ and $f_2=e^x\sin y$, and put $\textbf{a}=(0,\pi/3)$, $\textbf{b}=\textbf f(a)$. Find an explicity formula for the inverse of $\textbf{f}$, denoted $\textbf{...
Superunknown's user avatar
  • 2,749
1 vote
1 answer
71 views

Prove that $f$ doesn't attain its local maximum

Suppose that $f:\mathbb{R}^2\to\mathbb{R}^2$ is of class $C^3$ s.t $$\Delta f=\dfrac{\partial ^2f}{\partial x^2}+\dfrac{\partial ^2f}{\partial y^2}>0\quad\forall (x,y)\in\mathbb{R}^2.$$ Prove that ...
lee max's user avatar
  • 436
0 votes
1 answer
49 views

Why isn't the integral of the total differential of a function equal to the function? [duplicate]

For example let $w = 2x^3y$ then $$dw = \frac{\partial w}{\partial x}dx + \frac{\partial w}{\partial y}dy $$ $$\implies dw = 6x^2y\hspace{1.5mm} dx + 2x^3 \hspace{1.5mm} dy$$ Then why isn't it that $$...
Jack's user avatar
  • 11
0 votes
2 answers
29 views

Composition of a $L^2$ function with a $C^1$ monotone function (with some additional conditions) is again $L^2$

I don't know why this problem is giving me so much trouble but for whatever reason I cannot do this, maybe it's just too late in the day. Take $U \subseteq \mathbb R^n$ be open and bounded, $f: U \to \...
person's user avatar
  • 1,373
0 votes
0 answers
17 views

A second-order sufficient condition with a local condition for extreme value [closed]

Assume that f and g are twice continuously differentiable in a neighborhood of (x0, y0), g(x0, y0) = 0, (∇g)(x0, y0) ̸= 0 and (∇L)(x0, y0, λ0) = 0 for some λ0 ∈ R, where L(x, y, λ) = f(x, y)−λ * g (x, ...
Ákos Biszak's user avatar
2 votes
0 answers
21 views

Stokes theorem to calculate line integral

Let $\gamma$ be the intersection between $z=x^2+y^2$ and the plane $z=1+2x$. Calculate the work done by the field $F=(0,x,-y)$ when the curve $\gamma$ traverses on lap in positive direction seen from ...
per persson's user avatar
0 votes
0 answers
14 views

Why the limit by the level curve path c is always equals c? [closed]

i have a very begginer question, but i can't find the reason why the limit of any function of two variables when we take the level curve $f(x, y) = c$ path is equal c. I am thinking of this because, ...
flip141's user avatar
0 votes
0 answers
40 views

Calculus Identities

I am trying to write an expression to $\partial_t \|\nabla u\|_{L^p(\Omega)}^p.$ Here $\Omega$ is a smooth domain, the function $u$ has no regularity problems (you can take it smooth) and the normal ...
BGT_MATH's user avatar
  • 343
0 votes
0 answers
88 views

Inverse function theorem in the case of $\det=0$

Let $f:\mathbb{R}^2\to\mathbb{R}^2$ be defined by $$f(x,y)=(x^2-y^2, 2xy).$$ a) Does $f$ have local inverse function at $(0,0)$? b) Find the domain of $f$. My attempt: a) I just prove $f$ has local ...
Alex Nguyen's user avatar
  • 1,255
2 votes
2 answers
58 views

A problem using Chain Rule theorem

Let $F(u,v)$ be of class $C^1(\mathbb{R}^2)$ and $z=z(x,y)$ be the function defined by the equation $$F\left(x+\dfrac{z}{y},y+\dfrac{z}{x}\right)=0.$$ Prove that the function $z(x,y)$ satisfies the ...
Alex Nguyen's user avatar
  • 1,255
-1 votes
1 answer
38 views

Question about directional derivatives [closed]

The temperature of a $10\times 10$ inch hot plate is given by $T(x,y) = 100-x^2 - 2y^2$ at $x$ units to the right of the lower left and $y$ up from the lower left. If a bug on the plate can survive ...
Jason Benyousky's user avatar
1 vote
0 answers
52 views

How to find the maximum and minimum values, if any, of $G(x,y)=\frac{\sin(\pi \sqrt{x^2+y^2})}{\pi \sqrt{x^2+y^2}}?$

I have been trying to find the critical points of the function $G(x,y) = \frac{\sin(\pi \sqrt{x^2+y^2})}{\pi \sqrt{x^2+y^2}}$. The partial derivatives are $$G_{x}(x,y)=\frac{x}{\pi (x^2+y^2)^{3/2}} \...
limber's user avatar
  • 133
0 votes
1 answer
39 views

A point in a set is interior point iff

Definition of interior point: $x$ is interior point of $X\subset\mathbb{R}^n$ if there exists $\varepsilon >0$ such that $B(x,\epsilon )\subset X$. Let $A\subset\mathbb{R}^n$. $a\in A$ is an ...
user926356's user avatar
  • 1,296
0 votes
0 answers
33 views

What does it mean for an inclusion map to be smooth?

Sorry for the elementary question, but I do not understand what does it mean for an inclusion map to be "smooth"? My understanding is that an inclusion map will send an element $x$ into ...
Y2H's user avatar
  • 161
0 votes
0 answers
21 views

Conservative vector fields: curves and path [closed]

Suppose a vector field $F$ has the property that $$ \int_C F =0 $$ for all curves $C$ of class $C^1$. Is it true that the same property holds if we change $C$ for a path (yuxtaposition of $C^1$ curves,...
user210089's user avatar
0 votes
1 answer
104 views

Why $W$ is open in baby Rudin 2.28

9.28 Theorem: Let $f$ be a r $\mathscr{C}'$ -mapping of an open set $E$ $ \subset \mathbb{R}^{n+m}$ into $\mathbb{R}^n$, such that $f(a, b) = 0$ for some point $(a, b) \in E$. Put $A = f'(a, b)$ and ...
Mathematics enjoyer's user avatar
-2 votes
0 answers
21 views

Determining the Constant for Maximum Rate of Increase in a Multivariable Function Along a Given Vector [closed]

Let $f (x, y) = x^3 y − x y^2+ c x^2$ where $c$ is a constant. Find $c$ if $f$ increases fastest at the point $P_0 (3, 2)$ in the direction of the vector $\vec{v} = 2 \hat{i} + 5 \hat{j}$.
Edel's user avatar
  • 1
1 vote
0 answers
39 views

Generalization of mean value theorem?

For differentiable function $f: \mathbb{R} \to \mathbb{R}$ the mean value theorem states: given $a, b$ there exists $c$ such that $$ f(b) - f(a) = f'(c) (b-a). $$ Is there something similar for $f: \...
Johnny T.'s user avatar
  • 2,917
0 votes
1 answer
25 views

Conditions for Symmetry of mixed partials

I am studying symmetry of mixed partial derivatives in multivariate calculus(Clairaut's Theorem). Wikipedia says that differentiability of first order partials implies symmetry, to which I have found ...
xIxnx's user avatar
  • 3
2 votes
1 answer
31 views

If $F$ attains its local minimum at $x=0$ then $F(x)-F(0)\geq\xi x$

Let $F:\mathbb{R}\to\mathbb{R}$ be the convex mapping. Prove that there exists derivatives $F'(0+), F'(0-)$ and if $F$ attains its local minimum at $x=0$ then $$F(x)-F(0)\geq \xi x,\quad\forall x,\xi\...
lee max's user avatar
  • 436
0 votes
1 answer
42 views

Deal with discontinuity in double integrals

Let $f(x,y)= \frac{x^2+y^2}{x^2}$. Consider $$ \int_D f(x,y)$$ with $D=\{ (x,y): 0\leq y\leq x, x^2+y^2\leq 1 \}$ The function is unbounded in $D$, due to the denominator. I checked that cannot be ...
user210089's user avatar
-5 votes
0 answers
38 views

Suppose the temperature at (x,y,z) is given by M=xyz. In what direction could you go from (1,1,1) to keep the same temperature? [closed]

I’ve got this problem (which has been posed but I did not understand the answer to). I do not fully understand gradients and directional derivatives so could anyone point me in the right direction for ...
Jason Benyousky's user avatar

1
2 3 4 5
718